How to print answers only in exam?












0















With printanswers I can print all the answers following each of the questions. Now I want to print the answers only without the questions. I need this because I want to upload answers only, not the questions on website.



documentclass[11pt,oneside,A4paper,final,leqno]{exam}
usepackage{tikz}
usepackage{enumerate}
usepackage{amsfonts}
usepackage{amsmath}
usepackage{longtable}
usepackage{xcolor}
renewcommand{sectionmark}[1]{ {markright{rm smallthesection. #1}}}
usepackage[paper=a4paper,margin=1.086571in]{geometry}%
usepackage{etex}

usepackage[mathscr]{eucal}
usepackage{graphics, graphpap}
usepackage{multicol,color}
usepackage{indentfirst}

renewcommand{thefootnote}{}



usepackage{pgf,tikz}
usepackage{mathrsfs}
usetikzlibrary{arrows}

usepackage[absolute,overlay]{textpos}
usepackage{pgf,tikz}
usepackage{mathrsfs}
usetikzlibrary{arrows}
usepackage{amsthm}
defbi{textbf{i}}
defbj{textbf{j}}
defbk{textbf{k}}


usepackage[all]{xy} SelectTips{eu}{}

renewcommandquestionlabel{thequestion.} %replace period with parenthesis
renewcommand{thequestion}{bfseries arabic{question}}
printanswers
begin{document}


section*{Maths Test # 1901 rule{5.92cm}{.4pt}
}




begin{questions}



question What is the probability of picking an ace in two consecutive attempts in a 52 card deck?
begin{multicols}{4}
begin{choices}
choice $displaystylefrac{13}{221}$
choice $displaystylefrac{23}{221}$
choice $displaystylefrac{33}{221}$
choice $displaystylefrac{43}{221}$
end{choices}
end{multicols}
begin{solution} $(C)$.
In the first attempt, the chance not to get an ace is $48/52$. In the second one, the chance is $47/51$. Therefore, the chance not to get an ace in two consecutive attempts is $48*47/51*52=188/221$. Hence, the result must be $1-188/221=33/221$.
end{solution}




question Given three points $A(1,5), B(4,1)$ and $C(5,8)$. What is the angle $widehat{ACB}$?
begin{multicols}{4}
begin{choices}
choice $90$
choice $45$ %b
choice $60$
choice $120$
end{choices}
end{multicols}
begin{solution} $(B)$
We have $overrightarrow{AB}=(3,-4)$ and $overrightarrow{AC}=(4,3)$. Therefore, $ABC$ is an isosceles triangle with $widehat{BAC}=90$. Thus, the answer is $widehat{ACB}=45$.
end{solution}

question Suppose the ball moves freely inside the square domain with constant speed and the reflection off the boundary elastic and subject to a familiar law: the angle of incidence equals the angle of reflection. Put the ball at the center of the domain. At which angle $alpha$ does the ball have to start so that it will hit one of the four corners of the domain?
begin{center}

begin{tikzpicture}
clip(-4.5,1.5) rectangle (2.5,8.5);
%fill[line width=.4pt,fill=white] (2.,2.) -- (2.,8.) -- (-4.,8.) -- (-4.,2.) -- cycle;
draw[line width=.4pt,color=black] (-0.6928530532823249,2.) -- (-0.6928530532823249,2.3071469467176757) -- (-1.,2.3071469467176757) -- (-1.,2.) -- cycle;
draw [shift={(-1.,5.)},line width=.6pt,color=black] (0,0) -- (270.:1) arc (270.:303.6900675259798:1) -- cycle;
draw [line width=.6pt] (2.,2.)-- (2.,8.);
draw [line width=.66pt] (2.,8.)-- (-4.,8.);
draw [line width=.6pt] (-4.,8.)-- (-4.,2.);
draw [line width=.6pt] (-4.,2.)-- (2.,2.);
draw [line width=.6pt] (-1.,5.)-- (-1.,2.);
draw [line width=.6pt] (-1.,5.)-- (1.,2.);
begin{scriptsize}
draw (-0.1,4.6081570530868605) node {$alpha$};
end{scriptsize}
end{tikzpicture}
end{center}


begin{choices}
choice $displaystyletan{alpha}=frac{1}{2017}$
choice $displaystyletan{alpha}=frac{1}{2018}$ %b
choice $displaystyletan{alpha}=frac{2}{2019}$
choice $displaystyletan{alpha}=2020$
end{choices}


end{questions}

end{document}









share|improve this question



























    0















    With printanswers I can print all the answers following each of the questions. Now I want to print the answers only without the questions. I need this because I want to upload answers only, not the questions on website.



    documentclass[11pt,oneside,A4paper,final,leqno]{exam}
    usepackage{tikz}
    usepackage{enumerate}
    usepackage{amsfonts}
    usepackage{amsmath}
    usepackage{longtable}
    usepackage{xcolor}
    renewcommand{sectionmark}[1]{ {markright{rm smallthesection. #1}}}
    usepackage[paper=a4paper,margin=1.086571in]{geometry}%
    usepackage{etex}

    usepackage[mathscr]{eucal}
    usepackage{graphics, graphpap}
    usepackage{multicol,color}
    usepackage{indentfirst}

    renewcommand{thefootnote}{}



    usepackage{pgf,tikz}
    usepackage{mathrsfs}
    usetikzlibrary{arrows}

    usepackage[absolute,overlay]{textpos}
    usepackage{pgf,tikz}
    usepackage{mathrsfs}
    usetikzlibrary{arrows}
    usepackage{amsthm}
    defbi{textbf{i}}
    defbj{textbf{j}}
    defbk{textbf{k}}


    usepackage[all]{xy} SelectTips{eu}{}

    renewcommandquestionlabel{thequestion.} %replace period with parenthesis
    renewcommand{thequestion}{bfseries arabic{question}}
    printanswers
    begin{document}


    section*{Maths Test # 1901 rule{5.92cm}{.4pt}
    }




    begin{questions}



    question What is the probability of picking an ace in two consecutive attempts in a 52 card deck?
    begin{multicols}{4}
    begin{choices}
    choice $displaystylefrac{13}{221}$
    choice $displaystylefrac{23}{221}$
    choice $displaystylefrac{33}{221}$
    choice $displaystylefrac{43}{221}$
    end{choices}
    end{multicols}
    begin{solution} $(C)$.
    In the first attempt, the chance not to get an ace is $48/52$. In the second one, the chance is $47/51$. Therefore, the chance not to get an ace in two consecutive attempts is $48*47/51*52=188/221$. Hence, the result must be $1-188/221=33/221$.
    end{solution}




    question Given three points $A(1,5), B(4,1)$ and $C(5,8)$. What is the angle $widehat{ACB}$?
    begin{multicols}{4}
    begin{choices}
    choice $90$
    choice $45$ %b
    choice $60$
    choice $120$
    end{choices}
    end{multicols}
    begin{solution} $(B)$
    We have $overrightarrow{AB}=(3,-4)$ and $overrightarrow{AC}=(4,3)$. Therefore, $ABC$ is an isosceles triangle with $widehat{BAC}=90$. Thus, the answer is $widehat{ACB}=45$.
    end{solution}

    question Suppose the ball moves freely inside the square domain with constant speed and the reflection off the boundary elastic and subject to a familiar law: the angle of incidence equals the angle of reflection. Put the ball at the center of the domain. At which angle $alpha$ does the ball have to start so that it will hit one of the four corners of the domain?
    begin{center}

    begin{tikzpicture}
    clip(-4.5,1.5) rectangle (2.5,8.5);
    %fill[line width=.4pt,fill=white] (2.,2.) -- (2.,8.) -- (-4.,8.) -- (-4.,2.) -- cycle;
    draw[line width=.4pt,color=black] (-0.6928530532823249,2.) -- (-0.6928530532823249,2.3071469467176757) -- (-1.,2.3071469467176757) -- (-1.,2.) -- cycle;
    draw [shift={(-1.,5.)},line width=.6pt,color=black] (0,0) -- (270.:1) arc (270.:303.6900675259798:1) -- cycle;
    draw [line width=.6pt] (2.,2.)-- (2.,8.);
    draw [line width=.66pt] (2.,8.)-- (-4.,8.);
    draw [line width=.6pt] (-4.,8.)-- (-4.,2.);
    draw [line width=.6pt] (-4.,2.)-- (2.,2.);
    draw [line width=.6pt] (-1.,5.)-- (-1.,2.);
    draw [line width=.6pt] (-1.,5.)-- (1.,2.);
    begin{scriptsize}
    draw (-0.1,4.6081570530868605) node {$alpha$};
    end{scriptsize}
    end{tikzpicture}
    end{center}


    begin{choices}
    choice $displaystyletan{alpha}=frac{1}{2017}$
    choice $displaystyletan{alpha}=frac{1}{2018}$ %b
    choice $displaystyletan{alpha}=frac{2}{2019}$
    choice $displaystyletan{alpha}=2020$
    end{choices}


    end{questions}

    end{document}









    share|improve this question

























      0












      0








      0








      With printanswers I can print all the answers following each of the questions. Now I want to print the answers only without the questions. I need this because I want to upload answers only, not the questions on website.



      documentclass[11pt,oneside,A4paper,final,leqno]{exam}
      usepackage{tikz}
      usepackage{enumerate}
      usepackage{amsfonts}
      usepackage{amsmath}
      usepackage{longtable}
      usepackage{xcolor}
      renewcommand{sectionmark}[1]{ {markright{rm smallthesection. #1}}}
      usepackage[paper=a4paper,margin=1.086571in]{geometry}%
      usepackage{etex}

      usepackage[mathscr]{eucal}
      usepackage{graphics, graphpap}
      usepackage{multicol,color}
      usepackage{indentfirst}

      renewcommand{thefootnote}{}



      usepackage{pgf,tikz}
      usepackage{mathrsfs}
      usetikzlibrary{arrows}

      usepackage[absolute,overlay]{textpos}
      usepackage{pgf,tikz}
      usepackage{mathrsfs}
      usetikzlibrary{arrows}
      usepackage{amsthm}
      defbi{textbf{i}}
      defbj{textbf{j}}
      defbk{textbf{k}}


      usepackage[all]{xy} SelectTips{eu}{}

      renewcommandquestionlabel{thequestion.} %replace period with parenthesis
      renewcommand{thequestion}{bfseries arabic{question}}
      printanswers
      begin{document}


      section*{Maths Test # 1901 rule{5.92cm}{.4pt}
      }




      begin{questions}



      question What is the probability of picking an ace in two consecutive attempts in a 52 card deck?
      begin{multicols}{4}
      begin{choices}
      choice $displaystylefrac{13}{221}$
      choice $displaystylefrac{23}{221}$
      choice $displaystylefrac{33}{221}$
      choice $displaystylefrac{43}{221}$
      end{choices}
      end{multicols}
      begin{solution} $(C)$.
      In the first attempt, the chance not to get an ace is $48/52$. In the second one, the chance is $47/51$. Therefore, the chance not to get an ace in two consecutive attempts is $48*47/51*52=188/221$. Hence, the result must be $1-188/221=33/221$.
      end{solution}




      question Given three points $A(1,5), B(4,1)$ and $C(5,8)$. What is the angle $widehat{ACB}$?
      begin{multicols}{4}
      begin{choices}
      choice $90$
      choice $45$ %b
      choice $60$
      choice $120$
      end{choices}
      end{multicols}
      begin{solution} $(B)$
      We have $overrightarrow{AB}=(3,-4)$ and $overrightarrow{AC}=(4,3)$. Therefore, $ABC$ is an isosceles triangle with $widehat{BAC}=90$. Thus, the answer is $widehat{ACB}=45$.
      end{solution}

      question Suppose the ball moves freely inside the square domain with constant speed and the reflection off the boundary elastic and subject to a familiar law: the angle of incidence equals the angle of reflection. Put the ball at the center of the domain. At which angle $alpha$ does the ball have to start so that it will hit one of the four corners of the domain?
      begin{center}

      begin{tikzpicture}
      clip(-4.5,1.5) rectangle (2.5,8.5);
      %fill[line width=.4pt,fill=white] (2.,2.) -- (2.,8.) -- (-4.,8.) -- (-4.,2.) -- cycle;
      draw[line width=.4pt,color=black] (-0.6928530532823249,2.) -- (-0.6928530532823249,2.3071469467176757) -- (-1.,2.3071469467176757) -- (-1.,2.) -- cycle;
      draw [shift={(-1.,5.)},line width=.6pt,color=black] (0,0) -- (270.:1) arc (270.:303.6900675259798:1) -- cycle;
      draw [line width=.6pt] (2.,2.)-- (2.,8.);
      draw [line width=.66pt] (2.,8.)-- (-4.,8.);
      draw [line width=.6pt] (-4.,8.)-- (-4.,2.);
      draw [line width=.6pt] (-4.,2.)-- (2.,2.);
      draw [line width=.6pt] (-1.,5.)-- (-1.,2.);
      draw [line width=.6pt] (-1.,5.)-- (1.,2.);
      begin{scriptsize}
      draw (-0.1,4.6081570530868605) node {$alpha$};
      end{scriptsize}
      end{tikzpicture}
      end{center}


      begin{choices}
      choice $displaystyletan{alpha}=frac{1}{2017}$
      choice $displaystyletan{alpha}=frac{1}{2018}$ %b
      choice $displaystyletan{alpha}=frac{2}{2019}$
      choice $displaystyletan{alpha}=2020$
      end{choices}


      end{questions}

      end{document}









      share|improve this question














      With printanswers I can print all the answers following each of the questions. Now I want to print the answers only without the questions. I need this because I want to upload answers only, not the questions on website.



      documentclass[11pt,oneside,A4paper,final,leqno]{exam}
      usepackage{tikz}
      usepackage{enumerate}
      usepackage{amsfonts}
      usepackage{amsmath}
      usepackage{longtable}
      usepackage{xcolor}
      renewcommand{sectionmark}[1]{ {markright{rm smallthesection. #1}}}
      usepackage[paper=a4paper,margin=1.086571in]{geometry}%
      usepackage{etex}

      usepackage[mathscr]{eucal}
      usepackage{graphics, graphpap}
      usepackage{multicol,color}
      usepackage{indentfirst}

      renewcommand{thefootnote}{}



      usepackage{pgf,tikz}
      usepackage{mathrsfs}
      usetikzlibrary{arrows}

      usepackage[absolute,overlay]{textpos}
      usepackage{pgf,tikz}
      usepackage{mathrsfs}
      usetikzlibrary{arrows}
      usepackage{amsthm}
      defbi{textbf{i}}
      defbj{textbf{j}}
      defbk{textbf{k}}


      usepackage[all]{xy} SelectTips{eu}{}

      renewcommandquestionlabel{thequestion.} %replace period with parenthesis
      renewcommand{thequestion}{bfseries arabic{question}}
      printanswers
      begin{document}


      section*{Maths Test # 1901 rule{5.92cm}{.4pt}
      }




      begin{questions}



      question What is the probability of picking an ace in two consecutive attempts in a 52 card deck?
      begin{multicols}{4}
      begin{choices}
      choice $displaystylefrac{13}{221}$
      choice $displaystylefrac{23}{221}$
      choice $displaystylefrac{33}{221}$
      choice $displaystylefrac{43}{221}$
      end{choices}
      end{multicols}
      begin{solution} $(C)$.
      In the first attempt, the chance not to get an ace is $48/52$. In the second one, the chance is $47/51$. Therefore, the chance not to get an ace in two consecutive attempts is $48*47/51*52=188/221$. Hence, the result must be $1-188/221=33/221$.
      end{solution}




      question Given three points $A(1,5), B(4,1)$ and $C(5,8)$. What is the angle $widehat{ACB}$?
      begin{multicols}{4}
      begin{choices}
      choice $90$
      choice $45$ %b
      choice $60$
      choice $120$
      end{choices}
      end{multicols}
      begin{solution} $(B)$
      We have $overrightarrow{AB}=(3,-4)$ and $overrightarrow{AC}=(4,3)$. Therefore, $ABC$ is an isosceles triangle with $widehat{BAC}=90$. Thus, the answer is $widehat{ACB}=45$.
      end{solution}

      question Suppose the ball moves freely inside the square domain with constant speed and the reflection off the boundary elastic and subject to a familiar law: the angle of incidence equals the angle of reflection. Put the ball at the center of the domain. At which angle $alpha$ does the ball have to start so that it will hit one of the four corners of the domain?
      begin{center}

      begin{tikzpicture}
      clip(-4.5,1.5) rectangle (2.5,8.5);
      %fill[line width=.4pt,fill=white] (2.,2.) -- (2.,8.) -- (-4.,8.) -- (-4.,2.) -- cycle;
      draw[line width=.4pt,color=black] (-0.6928530532823249,2.) -- (-0.6928530532823249,2.3071469467176757) -- (-1.,2.3071469467176757) -- (-1.,2.) -- cycle;
      draw [shift={(-1.,5.)},line width=.6pt,color=black] (0,0) -- (270.:1) arc (270.:303.6900675259798:1) -- cycle;
      draw [line width=.6pt] (2.,2.)-- (2.,8.);
      draw [line width=.66pt] (2.,8.)-- (-4.,8.);
      draw [line width=.6pt] (-4.,8.)-- (-4.,2.);
      draw [line width=.6pt] (-4.,2.)-- (2.,2.);
      draw [line width=.6pt] (-1.,5.)-- (-1.,2.);
      draw [line width=.6pt] (-1.,5.)-- (1.,2.);
      begin{scriptsize}
      draw (-0.1,4.6081570530868605) node {$alpha$};
      end{scriptsize}
      end{tikzpicture}
      end{center}


      begin{choices}
      choice $displaystyletan{alpha}=frac{1}{2017}$
      choice $displaystyletan{alpha}=frac{1}{2018}$ %b
      choice $displaystyletan{alpha}=frac{2}{2019}$
      choice $displaystyletan{alpha}=2020$
      end{choices}


      end{questions}

      end{document}






      pdftex






      share|improve this question













      share|improve this question











      share|improve this question




      share|improve this question










      asked 36 mins ago









      ThumboltThumbolt

      1,494821




      1,494821






















          0






          active

          oldest

          votes











          Your Answer








          StackExchange.ready(function() {
          var channelOptions = {
          tags: "".split(" "),
          id: "85"
          };
          initTagRenderer("".split(" "), "".split(" "), channelOptions);

          StackExchange.using("externalEditor", function() {
          // Have to fire editor after snippets, if snippets enabled
          if (StackExchange.settings.snippets.snippetsEnabled) {
          StackExchange.using("snippets", function() {
          createEditor();
          });
          }
          else {
          createEditor();
          }
          });

          function createEditor() {
          StackExchange.prepareEditor({
          heartbeatType: 'answer',
          autoActivateHeartbeat: false,
          convertImagesToLinks: false,
          noModals: true,
          showLowRepImageUploadWarning: true,
          reputationToPostImages: null,
          bindNavPrevention: true,
          postfix: "",
          imageUploader: {
          brandingHtml: "Powered by u003ca class="icon-imgur-white" href="https://imgur.com/"u003eu003c/au003e",
          contentPolicyHtml: "User contributions licensed under u003ca href="https://creativecommons.org/licenses/by-sa/3.0/"u003ecc by-sa 3.0 with attribution requiredu003c/au003e u003ca href="https://stackoverflow.com/legal/content-policy"u003e(content policy)u003c/au003e",
          allowUrls: true
          },
          onDemand: true,
          discardSelector: ".discard-answer"
          ,immediatelyShowMarkdownHelp:true
          });


          }
          });














          draft saved

          draft discarded


















          StackExchange.ready(
          function () {
          StackExchange.openid.initPostLogin('.new-post-login', 'https%3a%2f%2ftex.stackexchange.com%2fquestions%2f480826%2fhow-to-print-answers-only-in-exam%23new-answer', 'question_page');
          }
          );

          Post as a guest















          Required, but never shown

























          0






          active

          oldest

          votes








          0






          active

          oldest

          votes









          active

          oldest

          votes






          active

          oldest

          votes
















          draft saved

          draft discarded




















































          Thanks for contributing an answer to TeX - LaTeX Stack Exchange!


          • Please be sure to answer the question. Provide details and share your research!

          But avoid



          • Asking for help, clarification, or responding to other answers.

          • Making statements based on opinion; back them up with references or personal experience.


          To learn more, see our tips on writing great answers.




          draft saved


          draft discarded














          StackExchange.ready(
          function () {
          StackExchange.openid.initPostLogin('.new-post-login', 'https%3a%2f%2ftex.stackexchange.com%2fquestions%2f480826%2fhow-to-print-answers-only-in-exam%23new-answer', 'question_page');
          }
          );

          Post as a guest















          Required, but never shown





















































          Required, but never shown














          Required, but never shown












          Required, but never shown







          Required, but never shown

































          Required, but never shown














          Required, but never shown












          Required, but never shown







          Required, but never shown







          Popular posts from this blog

          サソリ

          広島県道265号伴広島線

          Setup Asymptote in Texstudio